K
Khách

Hãy nhập câu hỏi của bạn vào đây, nếu là tài khoản VIP, bạn sẽ được ưu tiên trả lời.

2 tháng 8 2019

pt \(\Leftrightarrow\)\(x^4+2x^2y^2+y^4=2y^2-x^2+3\)

\(\Leftrightarrow\)\(\left(x^2+y^2\right)^2-2\left(x^2+y^2\right)+1=-3x^2+4\)

\(\Leftrightarrow\)\(\left(x^2+y^2-1\right)^2=-3x^2+4\le4\)

\(\Rightarrow\)\(-1\le x^2+y^2\le3\)

29 tháng 1 2020

Ta có: \(2\left(x^2+y^2\right)=1+xy\)

\(\Leftrightarrow x^2+y^2=\frac{1+xy}{2}\)

\(P=7\left(x^4+y^4\right)+4x^2y^2\)

\(=7x^4+7y^4+4x^2y^2\)

\(\Rightarrow P=28x^3+28y^3+16xy\)

\(\Leftrightarrow P=0\Leftrightarrow28x^3+28y^3+16xy=0\)

\(\Leftrightarrow\orbr{\begin{cases}x=3\\y=4\end{cases}}\)

\(\Rightarrow P_{Min}=15\) và \(Max_P=\frac{12}{33}\)

NV
9 tháng 4 2021

\(\left(x-1;y-1\right)=\left(a;b\right)\Rightarrow\left\{{}\begin{matrix}a;b>0\\a+b\le2\end{matrix}\right.\)

\(A=\dfrac{\left(a+1\right)^4}{b^2}+\dfrac{\left(b+1\right)^4}{a^2}\ge\dfrac{1}{2}\left[\dfrac{\left(a+1\right)^2}{b}+\dfrac{\left(b+1\right)^2}{a}\right]^2\)

\(A\ge\dfrac{1}{2}\left[\dfrac{\left(a+b+2\right)^2}{a+b}\right]^2\ge\dfrac{1}{2}\left[\dfrac{8\left(a+b\right)}{a+b}\right]^2=32\)

 

AH
Akai Haruma
Giáo viên
29 tháng 3 2021

Lời giải:

Đặt $xy=t$

Áp dụng BĐT AM_GM:

$xy\leq \frac{(x+y)^2}{4}=3$. Như vậy $0\leq t\leq 3$

Ta có:

$P=(x^4+1)(y^4+1)=x^4y^4+x^4+y^4+1$

$=x^4y^4+(x^2+y^2)^2-2x^2y^2+1$

$=x^4y^4+[(x+y)^2-2xy]^2-2x^2y^2+1$

$=x^4y^4+2x^2y^2-48xy+145$

$=t^4+2t^2-48t+145$

$=t(t^3+2t-48)+145$

Vì $0\leq t\leq 3$ nên $t(t^3+2t-48)\leq 0$

$\Rightarrow P\leq 145$

Vậy $P_{\max}=145$. Giá trị này đạt tại $(x,y)=(0,2\sqrt{3})$ và hoán vị.

23 tháng 1 2021

1) Áp dụng bất đẳng thức AM - GM và bất đẳng thức Schwarz:

\(P=\dfrac{1}{a}+\dfrac{1}{\sqrt{ab}}\ge\dfrac{1}{a}+\dfrac{1}{\dfrac{a+b}{2}}\ge\dfrac{4}{a+\dfrac{a+b}{2}}=\dfrac{8}{3a+b}\ge8\).

Đẳng thức xảy ra khi a = b = \(\dfrac{1}{4}\).

NV
23 tháng 1 2021

2.

\(4=a^2+b^2\ge\dfrac{1}{2}\left(a+b\right)^2\Rightarrow a+b\le2\sqrt{2}\)

Đồng thời \(\left(a+b\right)^2\ge a^2+b^2\Rightarrow a+b\ge2\)

\(M\le\dfrac{\left(a+b\right)^2}{4\left(a+b+2\right)}=\dfrac{x^2}{4\left(x+2\right)}\) (với \(x=a+b\Rightarrow2\le x\le2\sqrt{2}\) )

\(M\le\dfrac{x^2}{4\left(x+2\right)}-\sqrt{2}+1+\sqrt{2}-1\)

\(M\le\dfrac{\left(2\sqrt{2}-x\right)\left(x+4-2\sqrt{2}\right)}{4\left(x+2\right)}+\sqrt{2}-1\le\sqrt{2}-1\)

Dấu "=" xảy ra khi \(x=2\sqrt{2}\) hay \(a=b=\sqrt{2}\)

3. Chia 2 vế giả thiết cho \(x^2y^2\)

\(\dfrac{1}{x}+\dfrac{1}{y}=\dfrac{1}{x^2}+\dfrac{1}{y^2}-\dfrac{1}{xy}\ge\dfrac{1}{4}\left(\dfrac{1}{x}+\dfrac{1}{y}\right)^2\)

\(\Rightarrow0\le\dfrac{1}{x}+\dfrac{1}{y}\le4\)

\(A=\left(\dfrac{1}{x}+\dfrac{1}{y}\right)\left(\dfrac{1}{x^2}+\dfrac{1}{y^2}-\dfrac{1}{xy}\right)=\left(\dfrac{1}{x}+\dfrac{1}{y}\right)^2\le16\)

Dấu "=" xảy ra khi \(x=y=\dfrac{1}{2}\)

27 tháng 10 2020

Bài 2: Ta có: x, y, z không âm và \(x+y+z=\frac{3}{2}\)nên \(0\le x\le\frac{3}{2}\Rightarrow2-x>0\)

Áp dụng bất đẳng thức AM - GM dạng \(ab\le\frac{\left(a+b\right)^2}{4}\), ta được: \(x+2xy+4xyz=x+4xy\left(z+\frac{1}{2}\right)\le x+4x.\frac{\left(y+z+\frac{1}{2}\right)^2}{4}=x+x\left(2-x\right)^2\)

Ta cần chứng minh \(x+x\left(2-x\right)^2\le2\Leftrightarrow\left(2-x\right)\left(x-1\right)^2\ge0\)*đúng*

Đẳng thức xảy ra khi \(\left(x,y,z\right)=\left(1,\frac{1}{2},0\right)\)

29 tháng 10 2020

Bài 3: Áp dụng đánh giá quen thuộc \(4ab\le\left(a+b\right)^2\), ta có: \(2\le\left(x+y\right)^3+4xy\le\left(x+y\right)^3+\left(x+y\right)^2\)

Đặt x + y = t thì ta được: \(t^3+t^2-2\ge0\Leftrightarrow\left(t-1\right)\left(t^2+2t+2\right)\ge0\Rightarrow t\ge1\)(dễ thấy \(t^2+2t+2>0\forall t\))

\(\Rightarrow x^2+y^2\ge\frac{\left(x+y\right)^2}{2}\ge\frac{1}{2}\)

\(P=3\left(x^4+y^4+x^2y^2\right)-2\left(x^2+y^2\right)+1=3\left[\frac{3}{4}\left(x^2+y^2\right)^2+\frac{1}{4}\left(x^2-y^2\right)^2\right]-2\left(x^2+y^2\right)+1\ge\frac{9}{4}\left(x^2+y^2\right)^2-2\left(x^2+y^2\right)+1\)\(=\frac{9}{4}\left[\left(x^2+y^2\right)^2+\frac{1}{4}\right]-2\left(x^2+y^2\right)+\frac{7}{16}\ge\frac{9}{4}.2\sqrt{\left(x^2+y^2\right)^2.\frac{1}{4}}-2\left(x^2+y^2\right)+\frac{7}{16}=\frac{9}{4}\left(x^2+y^2\right)-2\left(x^2+y^2\right)+\frac{7}{16}=\frac{1}{4}\left(x^2+y^2\right)+\frac{7}{16}\ge\frac{1}{8}+\frac{7}{16}=\frac{9}{16}\)Đẳng thức xảy ra khi x = y = 1/2